The equation of a circle is x2 + y2 + 2x - 10y - 38 = 0. What are the coordinates of the
center and the length of the radius of the circle?

Answers

Answer 1

Answer:

Centre = (-1, 5)

radius = 8units

Explanation

The standard form of the equation of a circle is expressed as

x^2+y^2+2gx+2fy+C = 0

Center = (-g, -f)

radius = √g²+f²-C

Given the equation of a circle is x² + y²+ 2x - 10y - 38 = 0.

On comparing

2gx = 2x

2g = 2

g = 2/2

g = 1

2fy = -10y

2f =-10

f =-5

Centre = (-1, -(-5)) = (-1, 5)

radius  = √1²+5²-(-38)

radius = √1+25+38

radius = √64

radius = 8units


Related Questions

if f(x) = 3x + 2, what is f(5)?​

Answers

Answer:

f(x)=3x+2 what is f(5)

f(5)=3(5)+2 = 17

Step-by-step explanation:

Hi there!  

»»————- ★ ————-««

I believe your answer is:  

[tex]f(5)=17[/tex]

»»————- ★ ————-««  

Here’s why:  

Replace the 'x' with the given value (in this case 5) and solve.

⸻⸻⸻⸻

[tex]\boxed{\text{Calculating the Answer....}}\\\\\rightarrow f(x)=3x+2; f(5)\\-------------\\\rightarrow f(5)=3(5)+2\\\\\rightarrow f(5)=15+2\\\\\rightarrow \boxed{f(5)=17}}[/tex]

⸻⸻⸻⸻

»»————- ★ ————-««  

Hope this helps you. I apologize if it’s incorrect.  

Write the equation of the line that passes through the points (-6,5) and (3,−5). Put your answer in fully reduced point-slope form, unless it is a vertical or horizontal line.

Answers

Answer:

[tex]\displaystyle y-5=-\frac{10}{9}(x+6)[/tex]

Or:

[tex]\displaystyle y+5=-\frac{10}{9}(x-3)[/tex]

Step-by-step explanation:

We want to write the equation of a line that passes through the points (-6, 5) and (3, -5) in point-slope form.

Point-slope form is given by:

[tex]y-y_1=m(x-x_1)[/tex]

Thus, first, we need to find the slope. We can use the slope formula:

[tex]\displaystyle m=\frac{\Delta y}{\Delta x}=\frac{(-5)-(5)}{(3)-(-6)}=\frac{-10}{9}=-\frac{10}{9}[/tex]

Next, we can use either of the two given points. I'll use (-6, 5). So, let (-6, 5) be (x₁, y₁). Substitute:

[tex]\displaystyle y-(5)=-\frac{10}{9}(x-(-6))[/tex]

Or, fully simplified:

[tex]\displaystyle y-5=\frac{-10}{9}(x+6)[/tex]

Using the other point, we will acquire:

[tex]\displaystyle y-(-5)=-\frac{10}{9}(x-(3))[/tex]

Or, simplified:

[tex]\displaystyle y+5=-\frac{10}{9}(x-3)[/tex]

Two complementary angles have measures 4x and x + 50. Find x.

Answers

Answer:

x =17

Step-by-step explanation:

Complementary angles add to 90

4x+ x+5 = 90

Combine like terms

5x+5 = 90

Subtract 5 from each side

5x+5-5 = 90-5

5x = 85

Divide by 5

5x/5 = 85/5

x =17

Which function graph is shown

Answers

Answer:

A) y=3cos(-x)

Step-by-step explanation:

Because the amplitude of the graph is 3, this eliminates options C and D as their amplitudes are different. With the period of the graph being 2π, B cannot be correct because that would have a period of π/2. Therefore, A is correct.

On a math exam, Joel had a score of 90, which was exactly 2 standard deviations above the mean. The standard deviation for the test is 3.

Answers

Answer:

84

Step-by-step explanation:

Score = 90

Standard deviation = 3

Score = mean + 2(standard deviation)

90 = mean + 2(3)

90 = mean + 6

Mean = 90 - 6

Mean = 84

What is the y-intercept for y = 5x^2 + 2x -2?

Answers

Answer: (0, -2) or -2

Step-by-step explanation:

Sub in 0 for x since at the y intercept x is 0

[tex]y=5(0)^2+2(0)-2\\y=5(0)+0-2\\y=0+0-2\\y=-2[/tex]

I need help asap pls pls pls...

Answers

Answer:

c)

Step-by-step explanation:

please don't trust ;-;

The answer to this question is C. The surface area of a sphere is S=4pi radius square. Where r=20cm so substitute it into the formula and you will get 1600pi cm^2

Sean and sarah spend $70 a month eating out how much money would they have after 15 years if they invested the money instead at 4 percent compounded monthly round your answer to the nearest hundredth

Answers

Answer:

Future Value= $17,170.12

Step-by-step explanation:

Giving the following information:

Monthly deposit (A)= $70

Interest rate (i)= 0.04/12= 0.0033

Number of periods (n)= 15*12= 180 months

To calculate the future value (amount saved), we need to use the following formula:

FV= {A*[(1+i)^n-1]}/i

A= monthly deposit

FV= {70*[(1.0033^180) - 1]} / 0.0033

FV= $17,170.12

The basketball team did not record how many baskets each player made during the last game. Zulie remembers that she made three
times as many baskets as Aileen. Melody knows that she made four more baskets than Aileen. Zoe thinks that she made 3 fewer baskets than Aileen.
Abigail is sure that she made the same number of baskets as Zoe. Altogether the five players made 82 baskets. How many baskets did each player
make?

Answers

Answer:

There is a mistake in the question. x results in a decimal answer, which is not possible for number of hoops.

Step-by-step explanation:

Aileen: x

Zulie: 3x

Melody: 4 + x

Zoe: x - 3

Abigail: x - 3

82 = (x) + (3x) + (4+x) + (x-3) + (x-3)

Combine Like Terms

x + 3x + x + x + x = 7x

4 - 3 - 3 = -2

7x - 2 = 82

7x = 80

x results in a decimal answer, which is not possible for number of hoops.

A population of mice is increasing exponentially. On Monday there were 120 most. One month later there were 132 most. Ready function tomorrow the situation. Determine the percent increase of the population of most of each month.

Answers

Answer:

[tex]y=120 * 1.1^x[/tex] --- function

The monthly rate is 10%

Step-by-step explanation:

Given

Let

[tex]x \to months[/tex]

[tex]y \to mice[/tex]

So, we have:

[tex](x_1,y_1) = (0,120)[/tex] --- Monday

[tex](x_2,y_2) = (1,132)[/tex] --- One month later

Required

The function

The function is represented as:

[tex]y=ab^x[/tex]

In [tex](x_1,y_1) = (0,120)[/tex], we have:

[tex]120 = a * b^0[/tex]

[tex]120 = a * 1[/tex]

[tex]120 = a[/tex]

[tex]a=120[/tex]

In [tex](x_2,y_2) = (1,132)[/tex], we have:

[tex]132 = a * b^1[/tex]

[tex]132 = a * b[/tex]

Substitute: [tex]a=120[/tex]

[tex]132 = 120 * b[/tex]

Solve for b

[tex]b = \frac{132}{120}[/tex]

[tex]b = 1.1[/tex]

So, the function is:

[tex]y=ab^x[/tex]

[tex]y=120 * 1.1^x[/tex]

To calculate the monthly rate (r), we have:

[tex]y =a(1 + r)^x[/tex]

Compare to: [tex]y =ab^x[/tex]

[tex]1 + r = b[/tex]

Make r the subject

[tex]r = b-1[/tex]

Substitute [tex]b = 1.1[/tex]

[tex]r = 1.1-1[/tex]

[tex]r = 0.1[/tex]

Express as percentage

[tex]r = 0.1*100\%[/tex]

[tex]r = 10\%[/tex]

Any help on this please ?

Answers

Answer:

x=35

Step-by-step explanation:

because the angles are next to each other they are supplementary, which makes the total of the angle 180 degrees.

the equation would be:

180=x+14 +4x -9

add the like terms

180=5x =5

then subtract 5 from both sides

175=5x

then divide five from both sides

x=35

A ball is dropped from a height of 5m. After each bounce it rises to 35% of its previous height. After how many bounces does the ball reach a Height of less than 50cm?

Answers

Answer:

After the third bounce the ball reaches a height of less than 50cm.

Step-by-step explanation:

Geometric sequence:

In a geometric sequence, the quotient between consecutive terms is always the same, and it's called common ratio. The nth term of a geometric sequence is given by:

[tex]A_n = A_0(r)^{n}[/tex]

In which [tex]A_0[/tex] is the first term and r is the common ratio.

A ball is dropped from a height of 5m.

This means that [tex]A_0 = 5[/tex]

After each bounce it rises to 35% of its previous height.

This means that [tex]r = 0.35[/tex]

Thus

[tex]A_n = A_0(r)^{n}[/tex]

[tex]A_n = 5(0.35)^{n}[/tex]

After how many bounces does the ball reach a Height of less than 50cm?

50cm = 0.5m. This is n for which [tex]A_n = 0.5[/tex]. Thus

[tex]A_n = 5(0.35)^{n}[/tex]

[tex]0.5 = 5(0.35)^{n}[/tex]

[tex](0.35)^n = \frac{0.5}{5}[/tex]

[tex](0.35)^n = 0.1[/tex]

[tex]\log{(0.35)^n} = \log{0.1}[/tex]

[tex]n\log{0.35} = \log{0.1}[/tex]

[tex]n = \frac{\log{0.1}}{\log{0.35}}[/tex]

[tex]n = 2.19[/tex]

Rounding up:

After the third bounce the ball reaches a height of less than 50cm.



Please help I will award brainless to whoever helps please it’s urgent

Answers

Answer:

128 sq units :)

Step-by-step explanation:

hope this helps! have a great day :)

20 Points!!!

A ping pong ball is released from a height of 60 centimeters (cm) and bounces to a height that is 3/4 the previous height. What function estimates the height, H, in cm of the ping pong ball after x bounces?

Enter a number in each empty box to correctly complete the function.

H = ____ (____)^x

Answers

Multiply the starting height (60) by the height of each bounce (3/4) raised to the number of bounces (x)

H = (60)(3/4)^x

The answer to the given expression is:

H = 60(3/4)^x

What do we mean by exponents?

A number is raised by another number which is called the power/exponent which implies that the number is multiplied by itself for that many number of times.

How do we solve for H?

After every release, it bounces 3/4th of the previously released height.

After 1st bounce H = 60(3/4)

After 2nd bounce, it will be 3/4th of the previous H,

∴ H = 60(3/4)(3/4) = 60(3/4)^2

Similarly, after the 3rd bounce, it will be 3/4th of the previous H,

∴ H = 60(3/4)^2 * (3/4) = 60(3/4)^3

So it goes on like this. After x bounces,

H = 60(3/4)^x

Learn more about the Exponents at

https://brainly.com/question/11464095

#SPJ2

hello i give points!



Describe how the data is dispersed in a box plot with the following data set: 25, 36, 21, 30, 20, 32, 38, 19, 36, 31, 26, 33, 27, 18, 24.

Answers

Answer:

i would say that the box plot will look somewhat in the middle between 18 and 36 with 36 being the largest and 18 as the smallest

the data will also wont have a major difference in them

hope this helps<3

Kara and Karl love steak, fried chicken, hamburgers, mashed potatoes, and french fries. They like green beans and peas. How many different meals including a meat, potatoes and a green vegetable can they make from these choices? 8 meals 16 meals a. C. b. 12 meals d. 24 meals​

Answers

Answer:

steak, chicken, and hamburger are meats. 3 meats.

The French fries and mashed potatoes are potatoes. 2 Potatoes.

And Green beans and peas are green vegetables. 2 Vegetables.

I think we just do 3 x 2 x 2 = 12 Meals.

hope that helps bby<3


I will give BRAINLIEST to the correct answer
Find the measure of angle 3

Answers

Answer:

2X+100 =8X+208

8X-2X=100-208

6X=-108

X=-18

8X+208=(8×-18)+208

=-144+208

=64

ANGLE 3=180-64

=116

Answer: The measurement of angle #3 on the photo you provided it would be a 300 Degree angle if that is what you're asking

Step-by-step explanation: If you're asking what measure of an angle #3 is the correct measurement is 300 Degrees

Given the linear equation 5x+3y-9=0. Write another linear equation in two variables such that the geometrical representation of the pair so formed are parallel lines​

Answers

Answer:[tex]5x+3y+1=0[/tex]

Step-by-step explanation:

Given

The linear equation is [tex]5x+3y-9=0[/tex]

The other linear equation parallel to the given line can be represented by

[tex]5x+3y+c=0[/tex]

Here, c can take any value . for example c=1, line becomes

[tex]\Rightarrow 5x+3y+1=0[/tex]

Como se encuentra la mediana?

Answers

Answer:

Para encontrar la mediana: Organice los puntos de datos de menor a mayor. Si el número de puntos de datos es impar, la mediana es el punto de datos intermedio de la lista. Si el número de puntos de datos es par, la mediana es el promedio de los dos puntos de datos intermedios de la lista.

Step-by-step explanation:

que tenga un lindo día

Share $150 among John, Mary, and Sarah so that each of the two girls receives twice as much as John. Calculate the amount of money Jon received.

Answers

They get 60 dollars each Sarah and Mary and John gets 30 dollars because 60+60=120 and add 30 and it is 150 and 30 is half of 60

Pls help I don’t know how to do this

Answers

Answer:

the answer is D

BECAUSE it satisfied the conditions

Given the following system of equations, what value of x makes the equations true?
x+y= 3.25
x-y= 1.75

Answers

x=2.5!! hope this helps
Answer: x=2.5 , y=0.75

Explanation:

x+y=3.25 => y=3.25-x ——(1)

x-y=1.75 ——(2)

Substitute equation (1) in (2) then,

x-(3.25-x)=1.75

x-3.25+x=1.75

2x-3.25=1.75

2x=1.75+3.25

2x=5

x=5/2=2.5

Substitute vale of x in equation (1) we get

y=3.25-2.5

y=0.75

Therefore, x=2.5 , y=0.75

Find the opposite of each polynomial.

Answers

Answer:

Step-by-step explanation:

The word "opposite" is indicated by the - sign out front. Mathematically, we multiply a -1 into the parenthesis which, not coincidentally, changes the signs of everything inside the parenthesis, making them "opposite". The opposite of this polynomial is

- a - b - c

answer please? a girl needs help like right now:))

Answers

Answer:

No

Step-by-step explanation:

ACD should be one of them CAE

Simplify the expression ( a^3/2)^3

Answers

Answer:

(a^9/8)

Step-by-step explanation:

(a/b)^n = (a^n/b^n)

is How many oranges did
Sally eat last month? a statistical question

Answers

Answer:

25

Step-by-step explanation:

Lets say it's a 30 day month. Sally really likes oranges so she eats them a lot.

please help me if you can. please show how you got the answer.

Answers

Answer:

Step-by-step explanation:

The area of a circle is A = πr², where r is the radius. That means in order to solve this we have to find the value of x, which is the diameter of the lake, and then divide it in half to get the radius. To find x we will use similar triangles and proportions. x is the height of the big triangle and 4.5 is the height of the smaller triangle; 15.3 + 7.4 is the hypotenuse of the big triangle and 7.4 is the hypotenuse of the smaller triangle. Setting up our proportion:

[tex]\frac{x}{4.5}=\frac{15.3+7.4}{7.4}[/tex] which simplifies a bit to

[tex]\frac{x}{4.5}=\frac{22.7}{7.4}[/tex] and cross multiply to solve for x:

7.4x = 102.15 so

x = 13.8  That is the diameter of the lake. Divide it in half to get 6.9, the radius. Applying the area formula for a circle:

A = (3.14)(6.9)² and

A = 3.14(47.61) so

A = 149.5 which rounds to 150, Choice C

Pls help me solve pls show how you got the answer

Answers

Answer:

48 length units

Step-by-step explanation:

area of each squre = 9

side length of each square = sqrt(9) = 3

Length of each side of grid = 4*3 = 12

Perimeter of grid = 4 sides = 4*12 = 48 length units

ANSWER:

48length units

hope this helps

if x and 5x are supplementary angles, find them​

Answers

Answer:345678098321

Step-by-step explanation: I think so lol

Answer:

x+5x=180( adj.suppl.ls)

6x=180

6x÷6=180÷6

x=30

WILL MARK BRAINILIEST
are the angles corresponding, alternate interior, alternate exterior, or constructive interior​

Answers

Answer:

maybe it is corresponding

Other Questions
E=kq/r^2 chng minh in th V=kq/r t mi lin h gia in trng E v in th V True or False: The most fertile agricultural land in North America is used to produce mostly wheat and corn. Toni quierea un restaurante esta noche.A vaO B. irC. fueO D. voyWhats the answer write a letter of conplaint to the municipal commissioner about the state of the road in your area refusing clearing the despairs regularly In at least one hundred words, explain why Animal Farm was NOT an initial success in Britain? due today pls help me 12. Flowers whose reproductive structures consist onlyof stamens would be able to produceA) fruits with seedsB) fruits without seedsC) pollenD) ovules please help me i have 5 minutes left: )! ill give Brainliest thanks Read the following example:Quentin knew if he didn't pass his math test, he wouldn'tget into college and would live in his parent's basement forthe rest of his life.Which logical fallacy does the example contain?A. Begging the questionB. Straw manC. Ad hominemD. Slippery slot find the value of x. what is the relationship of these 2 angles? set up and solve an equation Consider this expression. 6 x +24Which of the following are equivalent to the expression?Select the three equivalent expressions.A. 6(x +4)B. 6(x + 24)C. 3(12 + 2x)D. 2(3x + 12)E. 3 + 3 + 20 + 4F 4x + 12 + 2x + 12 What is the area of the parallelogram shown? A man's reproductive age starts at puberty and ends when he dies. A woman's reproductive age is between Menarche (when she begins ovulating) and Menopause (when she stops ovulating). However, she has plenty eggs way before Menarche and way after Menopause. Why, do you think, Mother-Nature has put a limit on the reproductive age of women, but not men To win baseballs Triple Crown in pitching, a player has to lead his league in what? Who currentlyholds the Triple Crown in pitching? Pls help ASAP!!!!!!!!!!! I NEED HELP IMMEDIATELY!!!Jaime had ten posters, but only five could fit on his closet door. How many different ways can he arrange the five posters out of the ten on his closet door? A. 252 B. 648 C. 6,048 D. 30,240 factorize for mey + 3y + 2-sin2x=0 Select the correct answer to identify the prepositions in each sentence.From our cottage on the hill, we could see the skyline of the city beyond.a. on, of, beyond b. From, on, of, beyondc. on, of, beyondd. From, on, of Evaluate e - 1/2 f when c = 15 and f =2 Which character from Gligamesh: A New English version is an example of a supernatural force who intervened? 1._____you ever been to U.Shavedidwere2.___you have a pen I canborrow?DoWillHave